subject
Business, 18.02.2020 20:08 sparksjesse2830

Mr. Smith has been considering the purchase of a mutual fund and has decided he wants to buy $8,000 of the fund but only has $4,000 to invest now. He asks you, his registered representative, to lend him the other $4,000 which he will repay in 2 months. Which of the following is correct about this situation?

[A] The rep may lend the customer the funds as long as the agreement is in writing.
[B] The rep should establish a margin account for the customer and buy the fund shares on margin.
[C] The rep must deny the customer's request for a loan.
[D] The rep may lend the customer funds with the firm's prior approval.

ansver
Answers: 3

Another question on Business

question
Business, 21.06.2019 12:30
Savvy sightseeing had beginning equity of $90,000; revenues of $144,000, expenses of $83,000, and dividends to stockholders of $10,800. there were no stockholder investments during the year. calculate ending equity.
Answers: 3
question
Business, 22.06.2019 00:20
Overspeculation and a decrease in consumer confidence are both leading factors of: ?
Answers: 1
question
Business, 22.06.2019 09:00
Your grandmother told you a dollar doesn't go as far as it used to. she says the purchasing power of a dollar is much lesser than it used to be. explain what she means. try and use and explain terms like inflation and deflation in your answer.
Answers: 1
question
Business, 22.06.2019 15:30
On january 15, the end of the first biweekly pay period of the year, north company’s payroll register showed that its employees earned $32,000 of sales salaries. withholdings from the employees’ salaries include fica social security taxes at the rate of 6.2%, fica medicare taxes at the rate of 1.45%, $3,000 of federal income taxes, $772 of medical insurance deductions, and $260 of union dues. no employee earned > $7,000 in this first period. prepare the journal entry to record north company’s january 15 (employee) payroll expenses and liabilities.
Answers: 3
You know the right answer?
Mr. Smith has been considering the purchase of a mutual fund and has decided he wants to buy $8,000...
Questions
question
Mathematics, 06.01.2021 01:40
question
Mathematics, 06.01.2021 01:40
question
Mathematics, 06.01.2021 01:40
question
Mathematics, 06.01.2021 01:40
question
Advanced Placement (AP), 06.01.2021 01:40
question
Computers and Technology, 06.01.2021 01:40
Questions on the website: 13722367